Dowód z przekształceniem liniowym

Przestrzenie wektorowe, bazy, liniowa niezależność, macierze.... Formy kwadratowe, twierdzenia o klasyfikacji...
Benny01
Użytkownik
Użytkownik
Posty: 1116
Rejestracja: 11 wrz 2015, o 19:18
Płeć: Mężczyzna
Lokalizacja: Górnicza Dolina
Podziękował: 74 razy
Pomógł: 115 razy

Dowód z przekształceniem liniowym

Post autor: Benny01 »

Nie byłem pewny w jakim dziale umieścić to zadanie, a że potrzebne mi ono było do rachunku różniczkowego to wrzucam tutaj.

Pokazać, że jeśli \(\displaystyle{ T:R^m \rightarrow R^n}\) jest przekształceniem liniowym, to istnieje taka liczba \(\displaystyle{ M}\), że \(\displaystyle{ |T(h)| \le M|h|}\) dla \(\displaystyle{ h \in R^m}\).
Ostatnio zmieniony 17 wrz 2016, o 07:16 przez Kaf, łącznie zmieniany 1 raz.
Powód: Temat umieszczony w złym dziale.
szw1710

Dowód z przekształceniem liniowym

Post autor: szw1710 »

Niech \(\displaystyle{ h=\sum_{i=1}^m \alpha_ie_i}\) (rozwinięcie w bazie standardowej). Wtedy \(\displaystyle{ \left|T(h)\right|=\left|\sum_{i=1}^m\alpha_iT(e_i)\right|}\). Skorzystaj z nierówności Schwarza.
Benny01
Użytkownik
Użytkownik
Posty: 1116
Rejestracja: 11 wrz 2015, o 19:18
Płeć: Mężczyzna
Lokalizacja: Górnicza Dolina
Podziękował: 74 razy
Pomógł: 115 razy

Dowód z przekształceniem liniowym

Post autor: Benny01 »

\(\displaystyle{ \left| \sum_{i=1}^{m} \alpha _i T(e_i) \right| \le \left| \sum_{i=1}^{m} \alpha _i\right| \cdot \left| \sum_{i=1}^{m} T(e_i) \right|}\)
Co mi to daje?
Awatar użytkownika
AloneAngel
Użytkownik
Użytkownik
Posty: 630
Rejestracja: 19 mar 2012, o 17:57
Płeć: Mężczyzna
Lokalizacja: Kraków
Podziękował: 15 razy
Pomógł: 176 razy

Dowód z przekształceniem liniowym

Post autor: AloneAngel »

\(\displaystyle{ \left| \sum_{i=1}^{m} \alpha _i T(e_i) \right| \le \sum_{i=1}^{m} \left| \alpha_i \right| \left| T(e_i) \right| \le \sum_{i=1}^{m} \max_{k \in \{1, \ldots, m\}}{|\alpha_k|} \left| T(e_i) \right| = \max_{k \in \{1, \ldots, m\}}{|\alpha_k|} \sum_{i=1}^{m} | T(e_i) | = |h|_{\infty} \sum_{i=1}^{m} | T(e_i) |}\),
gdzie \(\displaystyle{ | . |_{\infty}}\) - norma maksimum. Niech \(\displaystyle{ M =\sum_{i=1}^{m} | T(e_i) |}\). Ponieważ w skończenie wymiarowych przestrzeniach wszystkie normy są równoważne więc masz, że \(\displaystyle{ | T(h)| \le |h| \cdot M \cdot c}\), dla dowolnej normy.
Benny01
Użytkownik
Użytkownik
Posty: 1116
Rejestracja: 11 wrz 2015, o 19:18
Płeć: Mężczyzna
Lokalizacja: Górnicza Dolina
Podziękował: 74 razy
Pomógł: 115 razy

Dowód z przekształceniem liniowym

Post autor: Benny01 »

Czemu norma z maksimum jest równa normie z \(\displaystyle{ h}\)?
Nie rozumiem pojęcia,
w skończenie wymiarowych przestrzeniach wszystkie normy są równoważne
Awatar użytkownika
AloneAngel
Użytkownik
Użytkownik
Posty: 630
Rejestracja: 19 mar 2012, o 17:57
Płeć: Mężczyzna
Lokalizacja: Kraków
Podziękował: 15 razy
Pomógł: 176 razy

Dowód z przekształceniem liniowym

Post autor: AloneAngel »

Znane Ci jest pojęcie równoważności norm? Bo jeżeli nie, to trzeba będzie to wykonać inaczej.
Benny01
Użytkownik
Użytkownik
Posty: 1116
Rejestracja: 11 wrz 2015, o 19:18
Płeć: Mężczyzna
Lokalizacja: Górnicza Dolina
Podziękował: 74 razy
Pomógł: 115 razy

Dowód z przekształceniem liniowym

Post autor: Benny01 »

Wydaje mi się że nie spotkałem się z takim pojęciem.
Awatar użytkownika
Premislav
Użytkownik
Użytkownik
Posty: 15687
Rejestracja: 17 sie 2012, o 13:12
Płeć: Mężczyzna
Lokalizacja: Warszawa
Podziękował: 196 razy
Pomógł: 5220 razy

Dowód z przekształceniem liniowym

Post autor: Premislav »

Moja propozycja (według oznaczeń Pana szw1710, z wyjątkiem podwójnej kreski na oznaczenie normy euklidesowej, bo nie chcę, żeby się myliło z wartością bezwzględną):

\(\displaystyle{ \left|\left| T(h)\right|\right|=\left|\left| \sum_{i=1}^{m}\alpha_iT(e_i) \right|\right| \le \sum_{i=1}^{m}|\alpha_i|\left|\left|T(e_i)\right|\right|}\)
- wynika to po prostu z nierówności trójkąta dla normy euklidesowej.

Następnie niech\(\displaystyle{ j \in \left\{ 1,\dots m\right\}}\) będzie takie, że
\(\displaystyle{ \left|\left|T(e_j)\right|\right|=\max_{1 \le i \le m}\left|\left|T(e_i)\right|\right|}\).
Wtedy \(\displaystyle{ \sum_{i=1}^{m}|\alpha_i|\left|\left|T(e_i)\right|\right| \le \left|\left|T(e_j)\right|\right| \sum_{i=1}^{m}|\alpha_i|}\)

Wreszcie zauważmy, że na mocy nierówności między średnią kwadratową a średnią arytmetyczną zachodzi:
\(\displaystyle{ \frac{1}{m} \sum_{i=1}^{m}|\alpha_i| \le \sqrt{\frac 1 m \sum_{i=1}^{m}\alpha_i^2 }= \frac{1}{\sqrt{m}}||h||}\),
a stąd i z powyższych nierówności wreszcie mamy
\(\displaystyle{ ||T(h)|| \le M ||h||}\)
dla \(\displaystyle{ M=\sqrt{m}||T(e_j)||}\),
gdzie \(\displaystyle{ e_j}\) jest wybrane jak wyżej.
Na pewno dało się dużo ładniej, ale nie pamiętam jak.
Benny01
Użytkownik
Użytkownik
Posty: 1116
Rejestracja: 11 wrz 2015, o 19:18
Płeć: Mężczyzna
Lokalizacja: Górnicza Dolina
Podziękował: 74 razy
Pomógł: 115 razy

Dowód z przekształceniem liniowym

Post autor: Benny01 »

Dzięki, na pewno bym tutaj nie pomyślał, żeby wrzucić tu średnie
Awatar użytkownika
Dasio11
Moderator
Moderator
Posty: 10218
Rejestracja: 21 kwie 2009, o 19:04
Płeć: Mężczyzna
Lokalizacja: Wrocław
Podziękował: 40 razy
Pomógł: 2361 razy

Dowód z przekształceniem liniowym

Post autor: Dasio11 »

Albo, tak jak na początku sugerował szw1710:

\(\displaystyle{ {\left\| T(h) \right\| = \left\| \sum_{i=1}^m \alpha_i \cdot T(e_i) \right\| \le \sum_{i=1}^m |\alpha_i| \cdot \| T(e_i) \| \le \sqrt{ \sum_{i=1}^m \alpha_i^2 } \cdot \sqrt{ \sum_{i=1}^m \| T(e_i) \|^2 } = M \cdot \| h \|,}}\)

gdzie

\(\displaystyle{ M = \sqrt{ \sum_{i=1}^m \| T(e_i) \|^2 }.}\)
szw1710

Dowód z przekształceniem liniowym

Post autor: szw1710 »

Kolega Premislav napisał prywatną wiadomość z pytaniem czemu usunąłem stąd pewien post. Owszem, był taki (w odpowiedzi na ten wpis), ale istniał może trzy minuty. Dlaczego? Bo rozumowanie nie działało w ogólności, a tylko dla funkcjonałów liniowych \(\displaystyle{ T:\RR^m\to\RR}\). Powtórzę więc. Otóż w takiej sytuacji mamy \(\displaystyle{ T(h)=\langle \alpha,h\rangle}\) dla dokładnie jednego \(\displaystyle{ \alpha}\). Stąd \(\displaystyle{ |T(h)|\le \|\alpha\|\cdot\|h\|}\) zgodnie z nierównością Schwarza. Co więcej, \(\displaystyle{ T\left(\frac{\alpha}{\|\alpha\|}\right)=\frac{\langle\alpha,\alpha\rangle}{\|\alpha\|}=\|\alpha\|}\), co oznacza, że \(\displaystyle{ \|T\|=\|\alpha\|}\). Proszę tu odwołać się do definicji normy odwzorowania liniowego. Oczywiście to rozumowanie stosuje się do funkcjonału niezerowego (\(\displaystyle{ \alpha\ne 0}\)), ale w tym przypadku nie ma czego dowodzić.

Zauważmy też, że przedstawione rozumowanie działa wszędzie tam, gdzie prawdziwe jest twierdzenie Riesza. Tak więc w roli dziedziny zamiast \(\displaystyle{ \RR^m}\) można wziąć dowolną przestrzeń Hilberta. Wtedy jednak trzeba dołożyć założenie ciągłości funkcjonału \(\displaystyle{ T}\). W przestrzeni skończenie wymiarowej jest ono w ciemno spełnione, gdyż tu wszystkie funkcjonały liniowe są ciągłe (tak naprawdę to właśnie wykazałem powyżej jak produkt uboczny).

W każdej nieskończenie wymiarowej przestrzeni unormowanej istnieje nieciągły funkcjonał liniowy. Wystarczy wziąć ciąg \(\displaystyle{ (x_n)}\) wektorów liniowo niezależnych i przenormować go, więc można od razu założyć, że \(\displaystyle{ \|x_n\|=1}\) dla każdego \(\displaystyle{ n}\). Uzupełniamy ten ciąg do bazy i bierzemy \(\displaystyle{ T(x_n)=n}\) oraz \(\displaystyle{ T(x)=0}\) dla pozostałych wektorów bazowych. Przedłużamy \(\displaystyle{ T}\) do odwzorowania liniowego na całą przestrzeń. Widać, że funkcjonał \(\displaystyle{ T}\) nie jest ograniczony na sferze jednostkowej, więc \(\displaystyle{ T}\) nie jest ciągły.
a4karo
Użytkownik
Użytkownik
Posty: 22204
Rejestracja: 15 maja 2011, o 20:55
Płeć: Mężczyzna
Lokalizacja: Bydgoszcz
Podziękował: 38 razy
Pomógł: 3753 razy

Dowód z przekształceniem liniowym

Post autor: a4karo »

Coś gdzieś pobłądziło. Mamy różne oszacowania \(\displaystyle{ |T(h)|}\): przez \(\displaystyle{ \sum|\alpha_i|}\), \(\displaystyle{ \max |\alpha_i|}\), \(\displaystyle{ \sqrt{\sum \alpha_i^2}}\), ale nigdzie nie ma oszacowania przez \(\displaystyle{ |h|}\).

Jasne, że z równoważności norm wynika, że i oszacowanie przez \(\displaystyle{ |h|}\) da sie zrobić, ale to troche naokoło.

Parę faktów:
1. odwzorowanie \(\displaystyle{ x\to T(x)}\) jest ciagłe.
2. Odwzorowanie \(\displaystyle{ y\to |y|}\) jest ciagłe
3. Sfera \(\displaystyle{ \{x: |x|=1\}}\) jest zwarta

(nawiasem mówiąc bez 2 i 3 równoważności norm sie chyba nie pokaże)
wniosek Istnieje \(\displaystyle{ \sup_{|x|=1}|T(x)|=M}\).

Stąd \(\displaystyle{ |T(x)|=\left||x|T\left(\frac{x}{|x|}\right)\right|\leq M|x|}\)
Awatar użytkownika
Premislav
Użytkownik
Użytkownik
Posty: 15687
Rejestracja: 17 sie 2012, o 13:12
Płeć: Mężczyzna
Lokalizacja: Warszawa
Podziękował: 196 razy
Pomógł: 5220 razy

Dowód z przekształceniem liniowym

Post autor: Premislav »

a4karo pisze:Coś gdzieś pobłądziło. Mamy różne oszacowania \(\displaystyle{ |T(h)|}\): przez \(\displaystyle{ \sum|\alpha_i|, \max |\alpha_i|, \sqrt{\sum \alpha_i^2}}\), ale nigdzie nie ma oszacowania przez \(\displaystyle{ |h|.}\)
Nie rozumiem, przecież tutaj zgodnie z założeniami \(\displaystyle{ h \in \RR^m}\), więc jeśli \(\displaystyle{ h= \sum_{i=1}^{m} \alpha_i e_i}\), gdzie \(\displaystyle{ e_i, i=1,\dots m}\) tworzą bazę standardową \(\displaystyle{ \RR^m}\), to \(\displaystyle{ |h|= \sqrt{ \sum_{i=1}^{m}\alpha_i^2 }}\). Mógłby Pan wyjaśnić, o co chodzi w tym zdaniu?
a4karo
Użytkownik
Użytkownik
Posty: 22204
Rejestracja: 15 maja 2011, o 20:55
Płeć: Mężczyzna
Lokalizacja: Bydgoszcz
Podziękował: 38 razy
Pomógł: 3753 razy

Dowód z przekształceniem liniowym

Post autor: a4karo »

Jeżeli \(\displaystyle{ |\codt|}\) jest dowolną normą, to wcale nie musi być \(\displaystyle{ |h|= \sqrt{ \sum_{i=1}^{m}\alpha_i^2 }}\)

Np: rozpatrz przestrzeń wielomianó stopnia \(\displaystyle{ \leq n}\) na odcinku \(\displaystyle{ [0,1]}\) i norme supremum . Potrafisz wyrazić \(\displaystyle{ |w|}\) przez współczynniki?
Awatar użytkownika
Premislav
Użytkownik
Użytkownik
Posty: 15687
Rejestracja: 17 sie 2012, o 13:12
Płeć: Mężczyzna
Lokalizacja: Warszawa
Podziękował: 196 razy
Pomógł: 5220 razy

Dowód z przekształceniem liniowym

Post autor: Premislav »

To oczywiste, ja to rozumiem. OK, dziękuję za wyjaśnienie tego zdania. No w sumie chyba przyjęliśmy, że skoro zadanie jest w dziale algebra liniowa, a nie analiza wyższa i funkcjonalna, to chodzi o standardową normę euklidesową w \(\displaystyle{ \RR^d}\), ale w sumie wcale nie jest to explicite napisane w treści, więc może to być niesłuszne.
ODPOWIEDZ